Những câu hỏi liên quan
Đẹp Trai Không Bao Giờ S...
Xem chi tiết
Đẹp Trai Không Bao Giờ S...
9 tháng 9 2019 lúc 22:12
Đỗ UYển dương
Xem chi tiết
Tuấn Nguyễn
6 tháng 7 2019 lúc 20:25

a) Ta có BĐT:

\(a^3+b^3=\left(a+b\right)\left(a^2+b^2-ab\right)\ge\left(a+b\right)ab\)

\(\Rightarrow a^3+b^3+abc\ge ab\left(a+b+c\right)\)

\(\Rightarrow\frac{1}{a^3+b^3+abc}\le\frac{1}{ab\left(a+b+c\right)}\)

Tương tự cho 2 bất đẳng thức còn lại rồi cộng theo vế:

\(VT\le\frac{1}{ab\left(a+b+c\right)}+\frac{1}{bc\left(a+b+c\right)}+\frac{1}{ca\left(a+b+c\right)}\)

\(=\frac{a+b+c}{abc\left(a+b+c\right)}=\frac{1}{abc}=VP\)

Khi \(a=b=c\)

Đỗ UYển dương
6 tháng 7 2019 lúc 20:26

cảm ơn ạ

quang phan duy
6 tháng 7 2019 lúc 20:37

câu 1 . Theo bđt côsi ta có \(a^3+b^3\ge ab(a+b)\)

\(\Rightarrow\frac{1}{a^3+b^3+abc}\le\frac{1}{ab(a+b)+abc}=\frac{1}{ab(a+b+c)}=\frac{c}{abc(a+b+c)}\)

tương tự \(\frac{1}{b^3+c^3+abc}\le\frac{a}{abc(a+b+c)}\)\(\frac{1}{a^3+c^3+abc}\le\frac{b}{abc(a+b+c)}\)

Cộng vế theo vế ta có  \(\frac{1}{b^3+c^3+abc}+\frac{1}{b^3+a^3+abc}+\frac{1}{a^3+c^3+abc}\le\frac{a+b+c}{abc(a+b+c)}=\frac{1}{abc}\)

\(\RightarrowĐPCM\)

hanvu
Xem chi tiết
Trần Phúc Khang
31 tháng 7 2019 lúc 19:53

1. BĐT ban đầu

<=> \(\left(\frac{1}{3}-\frac{b}{a+3b}\right)+\left(\frac{1}{3}-\frac{c}{b+3c}\right)+\left(\frac{1}{3}-\frac{a}{c+3a}\right)\ge\frac{1}{4}\)

<=>\(\frac{a}{a+3b}+\frac{b}{b+3c}+\frac{c}{c+3a}\ge\frac{3}{4}\)

<=> \(\frac{a^2}{a^2+3ab}+\frac{b^2}{b^2+3bc}+\frac{c^2}{c^2+3ac}\ge\frac{3}{4}\)

Áp dụng BĐT buniacoxki dang phân thức 

=> BĐT cần CM

<=> \(\frac{\left(a+b+c\right)^2}{a^2+b^2+c^2+3\left(ab+bc+ac\right)}\ge\frac{3}{4}\)

<=> \(a^2+b^2+c^2\ge ab+bc+ac\)luôn đúng 

=> BĐT được CM

Phùng Minh Quân
31 tháng 7 2019 lúc 21:15

2) \(a+b+c\le ab+bc+ca\le\frac{\left(a+b+c\right)^2}{3}\)\(\Leftrightarrow\)\(\left(a+b+c\right)^2-3\left(a+b+c\right)\ge0\)

\(\Leftrightarrow\)\(\left(a+b+c\right)\left(a+b+c-3\right)\ge0\)\(\Leftrightarrow\)\(a+b+c\ge3\)

ko mất tính tổng quát giả sử \(a\ge b\ge c\)

Có: \(3\le a+b+c\le ab+bc+ca\le3a^2\)\(\Leftrightarrow\)\(3a^2\ge3\)\(\Leftrightarrow\)\(a\ge1\)

=> \(\frac{1}{1+a+b}+\frac{1}{1+b+c}+\frac{1}{1+c+a}\le\frac{3}{1+2a}\le1\)

Dấu "=" xảy ra \(\Leftrightarrow\)\(a=b=c=1\)

Trần Phúc Khang
31 tháng 7 2019 lúc 21:46

Bạn @Diệu Linh@ làm nhầm dòng 5 rồi nhé

2, BĐT ban đầu 

<=> \(\left(1-\frac{1}{1+a+b}\right)+\left(1-\frac{1}{1+b+c}\right)+\left(1-\frac{1}{1+a+c}\right)\ge2\)

<=> \(\frac{\left(a+b\right)^2}{a+b+\left(a+b\right)^2}+\frac{\left(b+c\right)^2}{b+c+\left(b+c\right)^2}+\frac{\left(c+a\right)^2}{c+a+\left(c+a\right)^2}\ge2\)

Dùng BĐT buniacoxki dạng phân thức ở VT 

\(VT\ge\frac{4\left(a+b+c\right)^2}{2\left(a+b+c\right)+\left(a+b\right)^2+\left(b+c\right)^2+\left(c+a\right)^2}\)

Mà \(a+b+c\le ab+bc+ac\)

=> \(VT\ge\frac{4\left(a+b+c\right)^2}{2\left(ab+bc+ac\right)+2\left(a^2+b^2+c^2+ab+bc+ac\right)}=\frac{4\left(a+b+c\right)^2}{2\left(a+b+c\right)^2}=2\)(ĐPCM)

Dấu bằng xảy ra khi a=b=c=1

Anime Tổng Hợp
Xem chi tiết
Anime Tổng Hợp
20 tháng 2 2020 lúc 19:17

Ai giúp mình với :(( Mình cần gấp ạ

Khách vãng lai đã xóa
Thanh Tùng DZ
20 tháng 2 2020 lúc 20:20

Áp dụng BĐT Bu-nhi-a-cốp-ski,ta có :

\(\left(a^2+b^2+1^2\right)\left(1^2+1^2+c^2\right)\ge\left(a.1+b.1+c.1\right)^2=\left(a+b+c\right)^2\)

\(\Rightarrow\frac{1}{1+a^2+b^2}=\frac{1+1+c^2}{\left(a^2+b^2+1\right)\left(1+1+c^2\right)}\le\frac{2+c^2}{\left(a+b+c\right)^2}\)

Tương tự : \(\frac{1}{1+b^2+c^2}=\frac{1+1+a^2}{\left(1+b^2+c^2\right)\left(1+1+a^2\right)}\le\frac{2+a^2}{\left(a+b+c\right)^2}\)

  \(\frac{1}{1+c^2+a^2}=\frac{1+1+b^2}{\left(1+c^2+a^2\right)\left(1+1+b^2\right)}\le\frac{2+b^2}{\left(a+b+c\right)^2}\)

Cộng từng vế BĐT lại, ta được : 

\(\frac{1}{1+a^2+b^2}+\frac{1}{1+b^2+c^2}+\frac{1}{1+c^2+a^2}\le\frac{6+a^2+b^2+c^2}{\left(a+b+c\right)^2}=\frac{6+a^2+b^2+c^2}{a^2+b^2+c^2+2\left(ab+bc+ac\right)}=1\)

Vậy BĐT đã được chứng minh 

Khách vãng lai đã xóa
Anime Tổng Hợp
20 tháng 2 2020 lúc 20:34

Cảm ơn nhé!!

Khách vãng lai đã xóa
Yêu Toán
Xem chi tiết
Xuân Thái Hồ
Xem chi tiết
Khánh Vũ Trọng
Xem chi tiết
tth_new
7 tháng 8 2019 lúc 9:08

BĐT <=> \(\frac{2}{a^2+2}+\frac{2}{b^2+2}+\frac{2}{c^2+2}\le2\)

\(\Leftrightarrow1-\frac{a^2}{a^2+2}+1-\frac{b^2}{b^2+2}+1-\frac{c^2}{c^2+2}\le2\)

\(\Leftrightarrow\frac{a^2}{a^2+2}+\frac{b^2}{b^2+2}+\frac{c^2}{c^2+2}\ge1\)

Theo BĐT Svacxo:

\(VT\ge\frac{\left(a+b+c\right)^2}{a^2+b^2+c^2+6}=\frac{a^2+b^2+c^2+2\left(ab+bc+ca\right)}{a^2+b^2+c^2+6}=\frac{a^2+b^2+c^2+6}{a^2+b^2+c^2+6}=1\)

Vậy ta có đpcm.

P/s: Đúng ko ta?

bach nhac lam
Xem chi tiết
Ngô Bá Hùng
18 tháng 11 2019 lúc 20:54

1. Vai trò a, b, c như nhau. Không mất tính tổng quát. Giả sử \(a\ge b\ge0\)

\(ab+bc+ca=3\). Do đó \(ab\ge1\)

Ta cần chứng minh rằng \(\frac{1}{1+a^2}+\frac{1}{1+b^2}\ge\frac{2}{1+ab}\left(1\right)\)

\(\frac{2}{1+ab}+\frac{1}{1+c^2}\ge\frac{3}{2}\left(2\right)\)

Thật vậy: \(\left(1\right)\Leftrightarrow\frac{1}{1+a^2}-\frac{1}{1+ab}+\frac{1}{1+b^2}-\frac{1}{1+ab}\ge0\\ \Leftrightarrow\left(ab-a^2\right)\left(1+b^2\right)+\left(ab-b^2\right)\left(1+a^2\right)\ge0\\ \Leftrightarrow\left(a-b\right)\left[-a\left(1+b^2\right)+b\left(1+a^2\right)\right]\ge0\\ \Leftrightarrow\left(a-b\right)^2\left(ab-1\right)\ge0\left(BĐT:đúng\right)\)

\(\left(2\right)\Leftrightarrow c^2+3-ab\ge3abc^2\\ \Leftrightarrow c^2+ca+bc\ge3abc^2\Leftrightarrow a+b+c\ge3abc\)

BĐT đúng, vì \(\left(a+b+c\right)^2>3\left(ab+bc+ca\right)=q\)

\(ab+bc+ca\ge3\sqrt[3]{\left(abc\right)^2}\)

Nên \(a+b+c\ge3\ge3abc\)

Từ (1) và (2) ta có \(\frac{1}{1+a^2}+\frac{1}{1+b^2}+\frac{1}{1+c^2}\ge\frac{3}{2}\)

Dấu ''='' xảy ra \(\Leftrightarrow a=b=c=1\)

Khách vãng lai đã xóa
Ngô Bá Hùng
18 tháng 11 2019 lúc 21:16

Áp dụng BĐT Cauchy dạng \(\frac{9}{x+y+z}\le\frac{1}{x}+\frac{1}{y}+\frac{1}{z}\), ta được

\(\frac{9}{a+3b+2c}=\frac{1}{a+c+b+c+2b}\le\frac{1}{9}\left(\frac{1}{a+c}+\frac{1}{b+c}+\frac{1}{2b}\right)\)

Do đó ta được

\(\frac{ab}{a+3b+2c}\le\frac{ab}{9}\left(\frac{1}{a+c}+\frac{1}{b+c}+\frac{1}{2b}\right)=\frac{1}{9}\left(\frac{ab}{a+c}+\frac{ab}{b+c}+\frac{a}{2}\right)\)

Hoàn toàn tương tự ta được

\(\frac{bc}{2a+b+3c}\le\frac{1}{9}\left(\frac{bc}{a+b}+\frac{bc}{b+c}+\frac{b}{2}\right);\frac{ac}{3a+2b+c}\le\frac{1}{9}\left(\frac{ac}{a+b}+\frac{ac}{b+c}+\frac{c}{2}\right)\)

Cộng theo vế các BĐT trên ta được

\(\frac{ab}{a+3b+2c}+\frac{bc}{b+3c+2a}+\frac{ca}{c+3a+2b}\le\frac{1}{9}\left(\frac{ac+bc}{a+b}+\frac{ab+ac}{b+c}+\frac{bc+ab}{a+c}+\frac{a+b+c}{2}\right)=\frac{a+b+c}{6}\)Vậy BĐT đc CM

ĐẲng thức xảy ra khi và chỉ khi a = b = c >0

Khách vãng lai đã xóa
Akai Haruma
18 tháng 11 2019 lúc 22:38

Bài 2:

Áp dụng BĐT AM-GM:

\(a^2+2b^2+c^2=(a^2+b^2)+(a^2+c^2)\geq 2\sqrt{(a^2+b^2)(a^2+c^2)}\geq 2\sqrt{\frac{(a+b)^2}{2}.\frac{(a+c)^2}{2}}=(a+b)(a+c)\)

\(\Rightarrow \frac{ab^2}{a^2+2b^2+c^2}\leq \frac{ab^2}{(a+b)(a+c)}\)

Hoàn toàn tương tự với các phân thức còn lại:

\(\Rightarrow \text{VT}\leq \sum \frac{ab^2}{(a+b)(a+c)}=\frac{a^2b^2+b^2c^2+c^2a^2+abc(a+b+c)}{(a+b)(b+c)(c+a)}\)

Ta cần CM: \(\frac{a^2b^2+b^2c^2+c^2a^2+abc(a+b+c)}{(a+b)(b+c)(c+a)}\leq \frac{a+b+c}{4}\)

\(\Leftrightarrow 4(a^2b^2+b^2c^2+c^2a^2)+4abc(a+b+c)\leq (a+b+c)(a+b)(b+c)(c+a)\)

\(\Leftrightarrow 4(a^2b^2+b^2c^2+c^2a^2)+4abc(a+b+c)\leq (a+b+c)(a+b)(b+c)(c+a)\)

\(\Leftrightarrow 4(a^2b^2+b^2c^2+c^2a^2)+4abc(a+b+c)\leq (a+b+c)[(a+b+c)(ab+bc+ac)-abc]\)

\(\Leftrightarrow 2(a^2b^2+b^2c^2+c^2a^2)\leq (a^3b+ab^3)+(bc^3+b^3c)+(ca^3+c^3a)\)

(dễ thấy luôn đúng do theo BĐT AM-GM)

Do đó ta có đpcm.

Dấu "=" xảy ra khi $a=b=c$

Khách vãng lai đã xóa
Nguyễn Ngọc Linh Nhi
Xem chi tiết